Search found 194 matches


Statement 1 : Despite line a being perpendicular to line b, we may have a case (suppose when a and b make 45 degrees angle with the y axis)in which reflection of a is parallel to b and other cases in which reflection of a is not parallel to b. So, not sufficient. Statement 2 is not sufficient in its...

by The Iceman

Sat Feb 09, 2013 10:59 pm
Forum: Data Sufficiency
Topic: Difficult Question. Co Ordinate Geometry
Replies: 2
Views: 3267

If a number x is divided by another number y and the result is 102.15, then the remainder when x is divided by y is 0.15y. Also understand that the smallest possible natural number of the form 0.15y is 3; so our resulting remainder must be a multiple of 3. Among the answer options only 42 is a multi...

by The Iceman

Sat Feb 09, 2013 10:49 pm
Forum: Problem Solving
Topic: Remainder
Replies: 3
Views: 1976

Choosing r items out of a group of n items can be done in nCr ways, where nCr= n!/r!(n-r)!

In this problem we need to chose 3 members out of 10; so total possibilities= 10C3= 10!/(3!7!) = 120

by The Iceman

Sat Feb 09, 2013 10:42 pm
Forum: Problem Solving
Topic: Member
Replies: 1
Views: 380

Difference of the total scores after taking x+1 tests and x tests must equal 90.
=> 82(x+1) - 80 x = 90 => x =4

by The Iceman

Sat Feb 09, 2013 10:38 pm
Forum: Problem Solving
Topic: Test
Replies: 4
Views: 1068

Even multiples of 5 are basically multiples of 10. So, these numbers are 20,30,40,...100

The new sequence has the following numbers: 60,90,120...300

Since these numbers are evenly spaced, average of these numbers = (60+300)/2 = 180

by The Iceman

Sat Feb 09, 2013 12:07 pm
Forum: Problem Solving
Topic: average of new sequence
Replies: 2
Views: 1294

What is the sum of even multiples of 3 between 500-800 inclusive? A)193050 B)193047 C)32550 D)31836 E)31830 is even divisible is not same as even multiples?? will I not follow the same method that GMATGuruNY and Tommy mentioned? Evenly divisible by 3 means that the number is divisible by 3; however...

by The Iceman

Sat Feb 09, 2013 12:00 pm
Forum: Problem Solving
Topic: divisible by 3
Replies: 5
Views: 1946

For a certain examination, a score of 58 was 2 standard deviation below the mean and a score of 98 was 3 standard deviations above the mean. What was the mean score for the examination? A 74 B 76 C 78 D 80 E 82 Another perspective. Imagine this sentence appears as the last question on the GMAT and ...

by The Iceman

Thu Feb 07, 2013 7:55 am
Forum: Problem Solving
Topic: Mean, Standard deviation
Replies: 11
Views: 3427

Find, with justification, all positive integers n such that 2^2 + 2^5 + 2^n is a perfect square. Another one out of the tough rep. course, can't figure it out. Answers will be posted later. Couple of ways to interpret it. 2^2 + 2^5 + 2^n = 2^2{1 + 2.1.2^2 + 2^(n-2)} => n-2=4 =>n=6 2^2 + 2^5 + 2^n =...

by The Iceman

Thu Feb 07, 2013 4:07 am
Forum: Problem Solving
Topic: Tough problem
Replies: 3
Views: 1285

Both of the following idioms are correct:
so x that y
so x, so y that z

Option E correctly captures the intended meaning, using the latter construction.

by The Iceman

Thu Feb 07, 2013 2:59 am
Forum: Sentence Correction
Topic: 600+ Wind
Replies: 12
Views: 2899

For a certain examination, a score of 58 was 2 standard deviation below the mean and a score of 98 was 3 standard deviations above the mean. What was the mean score for the examination? A 74 B 76 C 78 D 80 E 82 Another perspective. Imagine this sentence appears as the last question on the GMAT and ...

by The Iceman

Thu Feb 07, 2013 2:50 am
Forum: Problem Solving
Topic: Mean, Standard deviation
Replies: 11
Views: 3427

Thanks Vineet, To find out f(7), we have to calculate it or we can apply the above formula again. Can you please explain giving one 3 digit number example? - nisagl ----------------------------- If I make mistakes please correct me. Basically f(1) to f(10) are easy to find. f(3)=6=> f(4)=4=> f(5)=2...

by The Iceman

Thu Feb 07, 2013 2:38 am
Forum: Problem Solving
Topic: Last two non zero digits of a factorial.
Replies: 7
Views: 8975

if Jon buys equal number of two types of mangoes at the rate of 9 mangoes per dollar and 11 mangoes per dollar, and then sell those at the rate of 10 mangoes per dollar, then what will be his profit or loss? I can solve it using shortcut method, but now want to learn about how its done actually in ...

by The Iceman

Wed Feb 06, 2013 11:55 pm
Forum: Problem Solving
Topic: profit or loss
Replies: 6
Views: 1407

This problem type is beyond the scope of GMAT, unless the problem involves a factorial of a single digit number (basically a very easy to calculate number). So, please do not invest time on such problems. However, solely for knowledge purposes I will give you a quick formula to calculate the last no...

by The Iceman

Wed Feb 06, 2013 11:21 pm
Forum: Problem Solving
Topic: Last two non zero digits of a factorial.
Replies: 7
Views: 8975

Hey IceMan, Thanks for the note. Indeed, I'm aware of absolute modifiers, but I didn't like this question because usually the added detail isn't quite so random. It tends to be a focusing detail. Of course, if this is an official GMAT question, then they're allowing it. Luckily for us, (B) is the b...

by The Iceman

Wed Feb 06, 2013 10:15 pm
Forum: Sentence Correction
Topic: :(
Replies: 7
Views: 1467

Hey Guys, I don't see the modifier in (B) being okay, sorry to say. Consider this example: I found my umbrella yesterday, its handle made of bone. I don't see that being okay. Is this a real question? What is the source? -t Tommy, this kind of modifier is known as an "absolute modifier" a...

by The Iceman

Tue Feb 05, 2013 11:16 pm
Forum: Sentence Correction
Topic: :(
Replies: 7
Views: 1467